Đến nội dung

DarkBlood nội dung

Có 618 mục bởi DarkBlood (Tìm giới hạn từ 06-05-2020)



Sắp theo                Sắp xếp  

#499839 Cho 3 số thực dương x, y, z thỏa mãn: $\frac{1}{1+x...

Đã gửi bởi DarkBlood on 18-05-2014 - 17:08 trong Bất đẳng thức và cực trị

Cho 3 số thực dương x, y, z thỏa mãn: $\frac{1}{1+x}+\frac{1}{1+y}+\frac{1}{1+z}\geq 2$

Tìm max P=xyz

Từ giả thiết suy ra $\dfrac{1}{1+x}\geq \dfrac{y}{1+y}+\dfrac{z}{1+z}\geq 2\sqrt{\dfrac{yz}{(1+y)(1+z)}}$

Chứng minh tương tự, ta có: 

$\dfrac{1}{(1+x)(1+y)(1+z)}\geq 8\sqrt{\dfrac{(xyz)^2}{\left [(1+x)(1+y)(1+z)  \right ]^2}}=\dfrac{8xyz}{(1+x)(1+y)(1+z)}$

Do đó $1\geq 8xyz \Leftrightarrow xyz\leq \dfrac{1}{8}$




#499103 a+b+c=1, tìm min $\frac{2ab}{c+ab}+\frac...

Đã gửi bởi DarkBlood on 14-05-2014 - 22:18 trong Bất đẳng thức - Cực trị

 

Bài toán 5. Cho các số dương a, b, c có tổng bẳng 1. Chứng minh rằng $\frac{a\left( b+c \right)}{4-9bc}+\frac{b\left( c+a \right)}{4-9ca}+\frac{c\left( a+b \right)}{4-9ab}\ge 6abc$

Ta có $\sum \dfrac{a(b+c)}{4-9bc}\geq 6abc \Leftrightarrow \sum \dfrac{b+c}{bc(4-9bc)}\geq 6$

Áp dụng bất đẳng thức $AM-GM$ cho $6$ số, ta có

$\sum \dfrac{b+c}{bc(4-9bc)}=\sum \left( \dfrac{1}{c(4-9bc)}+\dfrac{1}{b(4-9bc)} \right)\geq 6\sqrt[6]{\dfrac{1}{\left [ abc(4-9ab)(4-9bc)(4-9ca) \right ]^2}}$

Như vậy cần chứng minh $abc(4-9ab)(4-9bc)(4-9ca)\leq 1\Leftrightarrow 81abc(4-9ab)(4-9bc)(4-9ca)\leq 81$

Thật vậy, ta có

$81abc(4-9ab)(4-9bc)(4-9ca)\leq \left ( \dfrac{81abc+12-9ab-9bc-9ca}{4} \right )^4$

Mà $\dfrac{1}{a}+\dfrac{1}{b}+\dfrac{1}{c}\geq 9 \Leftrightarrow ab+bc+ca\geq 9abc \Leftrightarrow 81abc-9ab-9bc-9ca\leq 0$

Nên $81abc(4-9ab)(4-9bc)(4-9ca)\leq \left ( \dfrac{12}{4} \right )^4=81$

Vậy ra có điều phải chứng minh.

Đẳng thức xảy ra $\Leftrightarrow a=b=c=\dfrac{1}{3}$




#499100 a+b+c=1, tìm min $\frac{2ab}{c+ab}+\frac...

Đã gửi bởi DarkBlood on 14-05-2014 - 22:04 trong Bất đẳng thức - Cực trị

Bài toán 4. Cho các số dương a, b, c có tổng bẳng 1. Chứng minh rằng $\frac{2ab}{c+ab}+\frac{3bc}{a+bc}+\frac{2ca}{b+ca}\ge \frac{5}{3}$

Ta có:

$\frac{2ab}{c+ab}+\frac{3bc}{a+bc}+\frac{2ca}{b+ca}=\frac{2ab}{(c+a)(c+b)}+\frac{3bc}{(a+b)(a+c)}+\frac{2ca}{(b+c)(b+a)}=\dfrac{2ab(a+b)+3bc(b+c)+2ca(c+a)}{(a+b)(b+c)(c+a)}$

Do đó cần chứng minh

$6ab(a+b)+9bc(b+c)+6ca(c+a)\geq 5(a+b)(b+c)(c+a)$

$\Leftrightarrow 6ab(1-c)+9bc(1-a)+6ca(1-b)\geq 5(a+b+c)(ab+bc+ca)-5abc$

$\Leftrightarrow 6ab+9bc+6ca\geq 5(ab+bc+ca)+16abc$

$\Leftrightarrow ab+4bc+ca\geq 16abc$

$\Leftrightarrow \dfrac{1}{c}+\dfrac{4}{a}+\dfrac{1}{b}\geq 16$ $($Đúng theo BCS$)$

Đẳng thức xảy ra $\Leftrightarrow$ $a=\dfrac{1}{2}\ ;\ b=c=\dfrac{1}{4}$

 

 

Bài toán 11. Cho các số dương x, y, z có tích bẳng 8. Chứng minh rằng $\frac{{{x}^{2}}}{{{x}^{2}}+2x+4}+\frac{{{y}^{2}}}{{{y}^{2}}+2y+4}+\frac{{{z}^{2}}}{{{z}^{2}}+2z+4}\ge 1$

Đặt $x=\dfrac{2}{a}, y=\dfrac{2}{b}, z=\dfrac{2}{c}$ ta được bài toán 10.




#498752 Trận 9 - Bất đẳng thức

Đã gửi bởi DarkBlood on 13-05-2014 - 11:27 trong Thi giải toán Marathon cấp THCS 2014

Sách NCPT đã giới thiệu rồi mà sao phải chứng minh lại?

Chương trình cấp 2 là trong sách giáo khoa mà. 




#498717 Trận 9 - Bất đẳng thức

Đã gửi bởi DarkBlood on 13-05-2014 - 08:14 trong Thi giải toán Marathon cấp THCS 2014

Em nghĩ những bất đẳng thức như cô si 3 số, bunhiacopxki, BCS không có trong chương trình học cấp 2 nên cần phải chứng minh lại ạ.




#498155 Trận 9 - Bất đẳng thức

Đã gửi bởi DarkBlood on 09-05-2014 - 23:10 trong Thi giải toán Marathon cấp THCS 2014

Cho $x, y, z$ là các số dương thỏa mãn điều kiện $xyz = 1$. Tìm giá trị nhỏ nhất của biểu thức:  

$$E=\frac{1}{x^3(y+z)}+\frac{1}{y^3(z+x)}+\frac{1}{z^3(x+y)}.$$

Toán thủ ra đề: angleofdarkness

Mở rộng 1 của MSS 10

Cho số nguyên dương $n$ và các số dương $x, y, z$ thỏa mãn điều kiện $xyz=1.$ Tìm giá trị nhỏ nhất của biểu thức:

$$\textrm{A}=\dfrac{1}{x^{n+1}(y+z)}+\dfrac{1}{y^{n+1}(z+x)}+\dfrac{1}{z^{n+1}(x+y)}$$

 

Bài làm

 

Bổ đề 1: $($đã chứng minh$)$ Với mọi số thực $a, b, c, x, y, z$ $(x, y, z>0)$ ta luôn có:

$$\dfrac{a^2}{x}+\dfrac{b^2}{y}+\dfrac{c^2}{z}\geq \dfrac{(a+b+c)^2}{x+y+z}$$

 

Bồ đề 2: $($đã chứng minh$)$ Với mọi số dương $a, b, c,$ ta có

$$a^3+b^3+c^3\geq 3abc$$

 

Bổ đề 3: Với mọi số nguyên dương $n$ và các số dương $a, b, c,$ ta có

$$\dfrac{a^n}{b+c}+\dfrac{b^n}{c+a}+\dfrac{c^n}{a+b}\geq \dfrac{a^{n-1}+b^{n-1}+c^{n-1}}{2}$$

Chứng minh

Theo bổ đề 1 ta có $$\sum \dfrac{a^n}{b+c}=\sum \dfrac{a^{2(n-1)}}{a^{n-2}(b+c)}\geq \dfrac{(a^{n-1}+b^{n-1}+c^{n-1})^2}{\sum a^{n-2}(b+c)}$$

Đặt $n-2=k.$

Như vậy cần chứng minh $$\dfrac{a^{n-1}+b^{n-1}+c^{n-1}}{\sum a^{n-2}(b+c)}\geq \dfrac{1}{2}$$

$$\Leftrightarrow \dfrac{a^{k+1}+b^{k+1}+c^{k+1}}{\sum a^{k}(b+c)}\geq \dfrac{1}{2}$$

$$\Leftrightarrow 2(a^{k+1}+b^{k+1}+c^{k+1})\geq \sum a^{k}(b+c)$$

$$\Leftrightarrow (a^k-b^k)(a-b)+(a^k-c^k)(a-c)+(b^k-c^k)(b-c)\geq 0$$

Nếu $b\geq c>0$ thì $b^k-c^k\geq 0$ và $b-c\geq 0,$ suy ra $(b^k-c^k)(b-c)\geq 0$

 

Nếu $c\geq b>0$ thì $b^k-c^k\leq 0$ và $b-c\leq 0,$ suy ra $(b^k-c^k)(b-c)\geq 0$

 

Vậy $(b^k-c^k)(b-c)\geq 0$ với mọi $b, c>0$

 

Không mất tổng quát giả sử $a=\textrm{max}\left \{ a;\ b;\ c \right \}$

 

Suy ra $(a^k-b^k)(a-b)\geq 0$ và $(a^k-c^k)(a-c)\geq 0.$

 

Do đó $(a^k-b^k)(a-b)+(a^k-c^k)(a-c)+(b^k-c^k)(b-c)\geq 0$

 

Vậy ta có điều phải chứng minh. Đẳng thức xảy ra $\Leftrightarrow a=b=c.$

 

Quay lại bài toán

Đặt $x=\dfrac{1}{a}, y=\dfrac{1}{b}, z=\dfrac{1}{c}$ $(a,b,c>0)$

 

Vì $xyz=1$ nên $abc=1$

 

Khi đó $$\textrm{A}=\dfrac{a^{n+1}bc}{b+c}+\dfrac{ab^{n+1}c}{c+a}+\dfrac{abc^{n+1}}{a+b}=\dfrac{a^n}{b+c}+\dfrac{b^n}{c+a}+\dfrac{c^n}{a+b}$$

Áp dụng bổ đề 3, ta có $$\textrm{A}\geq \dfrac{a^{n-1}+b^{n-1}+c^{n-1}}{2}$$

Áp dụng bổ đề 2, ta có $$\textrm{A}\geq \dfrac{3\sqrt[3]{a^{n-1}.b^{n-1}.c^{n-1}}}{2}=\dfrac{3}{2}$$

Vậy $\textrm{min}\ \textrm{A}=\dfrac{3}{2}$ khi $a=b=c=1\Leftrightarrow x=y=z=1.$




#498132 Trận 9 - Bất đẳng thức

Đã gửi bởi DarkBlood on 09-05-2014 - 21:56 trong Thi giải toán Marathon cấp THCS 2014

Cho $x, y, z$ là các số dương thỏa mãn điều kiện $xyz = 1$. Tìm giá trị nhỏ nhất của biểu thức:  

$$E=\frac{1}{x^3(y+z)}+\frac{1}{y^3(z+x)}+\frac{1}{z^3(x+y)}.$$

Toán thủ ra đề: angleofdarkness

Bài làm của MSS 10

Bổ đề 1: Với mọi số thực $a, b, c, x, y, z$ $(x, y, z>0)$ ta luôn có:

$$\dfrac{a^2}{x}+\dfrac{b^2}{y}+\dfrac{c^2}{z}\geq \dfrac{(a+b+c)^2}{x+y+z}$$

Chứng minh

Trước hết ta chứng minh với mọi số thực $m, n, p, q$ $(p, q>0)$ ta có

$$\dfrac{m^2}{p}+\dfrac{n^2}{q}\geq \dfrac{(m+n)^2}{p+q} \ \ \ \ (\star)$$

Thật vậy bất đẳng thức $(\star)$ tương đương với $\dfrac{(mq-np)^2}{pq(p+q)}\geq 0$ $($Luôn đúng vì $p, q>0)$

Đẳng thức xảy ra $\Leftrightarrow \dfrac{m}{p}=\dfrac{n}{q}$

Áp dụng bất đẳng thức $(\star),$ ta có

$$\dfrac{a^2}{x}+\dfrac{b^2}{y}+\dfrac{c^2}{z}\geq \dfrac{(a+b)^2}{x+y}+\dfrac{c^2}{z}\geq \dfrac{(a+b+c)^2}{x+y+z}$$

Vậy ta có điều phải chứng minh. Đẳng thức xảy ra $\Leftrightarrow \dfrac{a}{x}=\dfrac{b}{y}=\dfrac{c}{z}$

 

Bổ đề 2: Với mọi số dương $a, b, c$ ta có

$$a^3+b^3+c^3\geq 3abc$$

Chứng minh

Bất đẳng thức đã cho tương đương với $\dfrac{1}{2}(a+b+c)[(a-b)^2+(b-c)^2+(c-a)^2]\geq 0$ (Luôn đúng với $a,b,c >0)$

Đẳng thức xảy ra $\Leftrightarrow a=b=c.$

 

Quay lại bài toán

Đặt $x=\dfrac{1}{a}, y=\dfrac{1}{b}, z=\dfrac{1}{c}$ $(a,b,c>0)$

 

Vì $xyz=1$ nên $abc=1$

 

Khi đó $$\textrm{E}=\dfrac{a^3bc}{b+c}+\dfrac{ab^3c}{c+a}+\dfrac{abc^3}{a+b}=\dfrac{a^2}{b+c}+\dfrac{b^2}{c+a}+\dfrac{c^2}{a+b}$$

Áp dụng bổ đề 1, ta có: $$\textrm{E}\geq \dfrac{(a+b+c)^2}{2(a+b+c)}=\dfrac{a+b+c}{2}$$

Áp dụng bổ đề 2, ta có: $$\textrm{E}=\dfrac{a+b+c}{2}\geq\dfrac{3\sqrt[3]{abc}}{2}=\dfrac{3}{2}$$

Đẳng thức xảy ra $\Leftrightarrow a=b=c=1 \Leftrightarrow x=y=z=1$

 

Vậy $\textrm{min}\ \textrm{E}=\dfrac{3}{2}$ khi $x=y=z=1.$

 

d = 10

S = 56




#496702 Chứng minh rằng:$\frac{a^{2}}{a+bc}+...

Đã gửi bởi DarkBlood on 02-05-2014 - 22:19 trong Bất đẳng thức và cực trị

 

Từ giả thiết $ab+bc+ca=abc$

Ta có

$$P=\sum \dfrac{a^3}{a^2+abc}=\sum \dfrac{a^3}{a^2+ab+bc+ca}=\sum \dfrac{a^3}{(a+b)(a+c)}$$

Áp dụng bất đẳng thức AM-GM, ta có:

$$\sum \dfrac{a^3}{(a+b)(a+c)}+\dfrac{1}{8} \sum \left ( a+b \right )+ \dfrac{1}{8} \sum \left ( a+c \right ) \geq \dfrac{3}{4} \sum a$$

$$\Leftrightarrow P\geq \dfrac{a+b+c}{4}$$

 



#496499 Chứng minh tứ giác $EIDB$ nội tiếp

Đã gửi bởi DarkBlood on 01-05-2014 - 22:10 trong Hình học

Cho đường tròn tâm $O$ đường kính $AB.$ Gọi $M$ là điểm đối xứng của $O$ qua $A.$ Qua $M$ kẻ đường thẳng cắt $(O)$ tại $C$ và $D.$ $AD$ cắt $BC$ tại $I.$ Gọi $E$ là trung điểm $AO.$ Chứng minh tứ giác $EIDB$ nội tiếp.




#495806 Chứng minh $A$ không phải là một số chính phương.

Đã gửi bởi DarkBlood on 28-04-2014 - 23:12 trong Số học

Bài toán: Chứng minh rằng với $m>3$ thì biểu thức $A=(m-1)^{4}-4m$ không thể viết dưới dạng một số chính phương.

-----------------------------------------------------------

$m>3$ nên $m\geq 4,$ do đó $2m(m-4)+1>0 \Rightarrow (m^2-2m)^2<A$

Lại có $A<(m-1)^4=(m^2-2m+1)^2$

Nên $(m^2-2m)^2<A<(m^2-2m+1)^2$

Từ đó có điều phải chứng minh.




#495599 Cho $\left | f(x) \right |\leq 1$ với mọi $x...

Đã gửi bởi DarkBlood on 27-04-2014 - 23:32 trong Bất đẳng thức và cực trị

Cho $f(x)=ax^2+bx+c$ với $a\neq 0$ sao cho $\left | f(x) \right |\leq 1$ với mọi $x$ thỏa $-1\leq x \leq 1.$ Chứng minh $4a^2+3b^2\leq 16.$




#494824 Phương trình $x^2-(m-1)^2x+m=0$ có các nghiệm đều nguyên.

Đã gửi bởi DarkBlood on 23-04-2014 - 23:17 trong Phương trình, hệ phương trình và bất phương trình

Tìm tất cả các số nguyên $m\geq 0$ sao cho phương trình
$$x^2-(m-1)^2x+m=0$$ có các nghiệm đều nguyên.




#493211 $$P=\dfrac{a^2}{a+bc}+\dfrac{b^2...

Đã gửi bởi DarkBlood on 15-04-2014 - 22:29 trong Bất đẳng thức và cực trị

Cho a,b,c là các số thực dương thỏa mãn:$\dfrac{1}{a}+\dfrac{1}{b}+\dfrac{1}{c}=1$.Chứng minh rằng:

$$P=\dfrac{a^2}{a+bc}+\dfrac{b^2}{b+ac}+\dfrac{c^2}{c+ab} \geq \dfrac{a+b+c}{4}$$

Từ giả thiết $ab+bc+ca=abc$

Ta có

$$P=\sum \dfrac{a^3}{a^2+abc}=\sum \dfrac{a^3}{a^2+ab+bc+ca}=\sum \dfrac{a^3}{(a+b)(b+c)}$$

Áp dụng bất đẳng thức AM-GM, ta có:

$$\sum \dfrac{a^3}{(a+b)(b+c)}+\dfrac{1}{8} \sum \left ( a+b \right )+ \dfrac{1}{8} \sum \left ( a+c \right ) \geq \dfrac{3}{4} \sum a$$

$$\Leftrightarrow P\geq \dfrac{a+b+c}{4}$$




#492295 Trận 7 - PT, HPT đại số

Đã gửi bởi DarkBlood on 11-04-2014 - 22:06 trong Thi giải toán Marathon cấp THCS 2014

Giải phương trình: $2x^{2}+5x-1=7\sqrt{x^{3}-1}}\ (\star)$

Đề thi của l4lzTeoz

Bài làm của MSS10: Em xin gửi lại bài làm

Cách 1:

ĐKXĐ: $x\geq 1$

 

Đặt $\sqrt{x-1}=a,\ \sqrt{x^2+x+1}=b.$ $(a\geq 0, b>0)$

 

Khi đó $\left\{\begin{matrix} 3a^2+2b^2=3x-3+2x^2+2x+2=2x^2+5x-1\\ ab=\sqrt{(x-1)(x^2+x+1)}=\sqrt{x^3-1} \end{matrix}\right.$

 

Phương trình đã cho trở thành

 

$3a^2+2b^2=7ab$

 

$\Leftrightarrow (3a-b)(a-2b)=0$

 

$\Leftrightarrow \begin{bmatrix} 3a=b\\ a=2b \end{bmatrix}$

 

 

Nếu $3a=b,$ ta có phương trình

 

$3\sqrt{x-1}=\sqrt{x^2+x+1}$

 

$\Leftrightarrow 9x-9=x^2+x+1$

 

$\Leftrightarrow x=4\pm\sqrt{6}\ (\textrm{TM})$

 

 

Nếu $a=2b,$ ta có phương trình

 

$\sqrt{x-1}=2\sqrt{x^2+x+1}$

 

$\Leftrightarrow x-1=4x^2+4x+4$ $($phương trình vô nghiệm vì $\Delta =-71<0)$

 

 

Vậy tập nghiệm của phương trình $(\star)$ là

 

$\boxed{S=\left \{ 4+\sqrt{6}\ ;\ 4+\sqrt{6} \right \}}$

 

 

Cách 2:

ĐKXĐ: $x\geq 1$

Ta có

$2x^{2}+5x-1=7\sqrt{x^{3}-1}$

$\Rightarrow (2x^2+5x-1)^2-49(x^3-1)=0$

$\Leftrightarrow (x^2-8x+10)(4x^2+3x+5)=0$

$\Leftrightarrow \begin{bmatrix} x^2-8x+10=0\\ 4x^2+3x+5=0 \end{bmatrix}$

 

Trường hợp 1: 

$x^2-8x+10=0$

$\Leftrightarrow x=4\pm\sqrt{6}\ (\textrm{TM})$

 

 

 

Trường hợp 2:

$4x^2+3x+5=0$ $($phương trình vô nghiệm vì $\Delta =-71<0)$

 

Vậy tập nghiệm của phương trình $(\star)$ là

$\boxed{S=\left \{ 4+\sqrt{6}\ ;\ 4+\sqrt{6} \right \}}$

 

  Hoan nghênh giải nhiều cách nhưng điểm vẫn vậy




#492276 Trận 7 - PT, HPT đại số

Đã gửi bởi DarkBlood on 11-04-2014 - 21:35 trong Thi giải toán Marathon cấp THCS 2014

Giải phương trình: $2x^{2}+5x-1=7\sqrt{x^{3}-1}\ (\star)$

Đề thi của l4lzTeoz

Bài làm của MSS10:

ĐKXĐ: $x\geq 1$

Đặt $\sqrt{x-1}=a,\ \sqrt{x^2+x+1}=b.$ $(a\geq 0, b>0)$

Khi đó $\left\{\begin{matrix} 3a^2+2b^2=3x-3+2x^2+2x+2=2x^2+5x-1\\ ab=\sqrt{(x-1)(x^2+x+1)}=\sqrt{x^3-1} \end{matrix}\right.$

Phương trình đã cho trở thành

$3a^2+2b^2=7ab$

$\Leftrightarrow (3a-b)(a-2b)=0$

$\Leftrightarrow \begin{bmatrix} 3a=b\\ a=2b \end{bmatrix}$

 

Nếu $3a=b,$ ta có phương trình

$3\sqrt{x-1}=\sqrt{x^2+x+1}$

$\Leftrightarrow 9x-9=x^2+x+1$

$\Leftrightarrow 4\pm\sqrt{6}\ (\textrm{TM})$

 

Nếu $a=2b,$ ta có phương trình

$\sqrt{x-1}=2\sqrt{x^2+x+1}$

$\Leftrightarrow x-1=4x^2+4x+4$ $($phương trình vô nghiệm vì $\Delta =-71<0)$

 

Vậy tập nghiệm của phương trình $(\star)$ là

$\boxed{S=\left \{ 4+\sqrt{6}\ ;\ 4+\sqrt{6} \right \}}$

 

 

      d =10

      S =17+10.3=47




#491960 Tìm $\textrm{Max}$ và $\textrm{Min...

Đã gửi bởi DarkBlood on 10-04-2014 - 19:09 trong Bất đẳng thức và cực trị

Cho $x, y, m$ là các số thực thỏa mãn

$$\left\{\begin{matrix} 2x-my=m\\ mx+y=\dfrac{3m^2+4}{m^2+4} \end{matrix}\right.$$

Tìm $\textrm{Max}$ và $\textrm{Min}$ của biểu thức $P=x^3+y^3.$




#491349 Tìm max của: $A=\frac{x}{4+2y+zx}+\frac...

Đã gửi bởi DarkBlood on 07-04-2014 - 23:38 trong Bất đẳng thức và cực trị

Cho $0<x \leq 2$ , $0<y \leq 2$, $0<z \leq 2$

Tìm max của:

$A=\frac{x}{4+2y+zx}+\frac{y}{4+2z+xy}+\frac{z}{4+2x+yz}$

Ta có:

Vì $x\leq 2\ ;\ z\leq 2$ nên $(x-2)(z-2)\geq 0 \Leftrightarrow xz\geq 2(x+z)-4$

Do đó $\dfrac{x}{4+2y+zx}\leq \dfrac{x}{2(x+y+z)}$

Chứng minh tương tự, suy ra $A\leq \dfrac{x+y+z}{2(x+y+z)}=\dfrac{1}{2}$

Vậy $\textrm{max}\ A=\dfrac{1}{2}$ khi $x=y=z=2$




#489501 Trận 6 - Phương trình nghiệm nguyên, đồng dư, chia hết

Đã gửi bởi DarkBlood on 29-03-2014 - 21:19 trong Thi giải toán Marathon cấp THCS 2014

Tồn tại hay không các cặp số nguyên (x,y) thỏa mãn phương trình sau đây ?

$$\sqrt{2025x^2+2012x+3188}=2013x-2011y+2094\ \ \ \ \ \ \ (1)$$

Đề của 

lenin1999

Bài làm của MSS 10:

Vì $x, y$ nguyên nên $2013x-2011y+2094 \in \mathbb{Z}$ 

Do đó $\sqrt{2025x^2+2012x+3188} \in \mathbb{N}$ $($vì $\sqrt{2025x^2+2012x+3188}\geq 0)$ 

 

Đặt $\sqrt{2025x^2+2012x+3188}=k$ $(k\in \mathbb{N})$

Khi đó $2025x^2+2012x+3188=k^2$

$\Leftrightarrow (2025x+1006)^2-(45k)^2=-5443664$

$\Leftrightarrow (2025x-45k+1006)(2025+45k+1006)=-5443664=-2^4.397.857\ \ \ \ \ \ (2)$

 

Vì $x \in \mathbb{Z}$ và $k \in \mathbb{N}$ nên $2025x-45k+1006\ ; 2025x+45k+1006 \in \mathbb{Z}$

Và $2025x-45k+1006\leq 2025x+45k+1006$

Mặt khác $(2025x-45k+1006)+(2025x+45k+1006)$ chia hết cho $2$ nên $2025x-45k+1006$ và $2025x+45k+1006$ cùng tính chẵn lẻ.

Lại có $2025x-45k+1006$ và $2025x-45k+1006$ đều chia $15$ dư $1$

 

Do đó phương trình $(1)$ tương đương với

$\left\{\begin{matrix} 2025x-45k+1006=-794\\ 2025x+45k+1006=6856 \end{matrix}\right. \Leftrightarrow \left\{\begin{matrix} x=1\\ k=85 \end{matrix}\right.$
 
Thay $x=1$ vào phương trình $(1)$ ta có phương trình
$\sqrt{2025+2012+3188}=2013-2011y+2094\Leftrightarrow y=2$
 
Vậy phương trình có nghiệm nguyên
$$(x;y)=(1;2)$$
 
 
   d =10
  S =17+10x3=47



#489020 Trận 3 - Hình học

Đã gửi bởi DarkBlood on 27-03-2014 - 11:26 trong Thi giải toán Marathon cấp THCS 2014

Bài này cũng bị lỗi hình vẽ mà anh 

BlackSelena

Diễn đàn hình như bị lỗi, em nộp lại hình cho bài của mình và anh tính điểm nha.
(Trừ nửa số điểm thì to quá @@)
Hình 1:

Hình 2:

 

Bài mình hiện giờ không có hình là do trang upanh mới ngừng hoạt động, còn lúc anh BlackSelena chấm bài thì vẫn có hình nha bạn :)




#488017 ĐỀ THI HSG LỚP 9 TỈNH PHÚ THỌ

Đã gửi bởi DarkBlood on 20-03-2014 - 22:34 trong Tài liệu - Đề thi

 

Câu 1:

b) Cho $p(x)=x^3-3x^2+14x-2$. Tìm các số tự nhiên $x< 100$ sao cho $p(x)\vdots 11$

 

Câu 4.

Cho đường tròn $(O;R)$ và dây cung $BC$ không đi qua tâm. Gọi $A$ là điểm chính giữa cung nhỏ $BC$. Gọi nội tiếp $\angle EAF$ quay quanh $A$ và cố định, sao cho $E,F$ nằm khác phía với $A$ so với $BC$, $AF,AE$ cắt $BC$ tại $M,N$. Lấy điểm $D$ sao cho tứ giác $MNED$ là hình bình hành.

a) Chứng minh : $MNEF$ là tứ giác nội tiếp

b) Gọi $I$ là tâm đường tròn ngoại tiếp $\Delta MDF$. Chứng minh : $I$ luôn thuộc 1 đường thẳng cố định khi góc nội tiếp $\angle EAF$ quay quanh $A$.

c) Tìm min của $OI$ khi $\angle EAF=60^{\circ},BC=R$

Câu 1b: $P(x)\vdots 11 \Rightarrow (x-1)^3\equiv\ 1\ (\bmod\ 11)$

Từ đây chứng minh được $x-1\equiv\ 1\ (\bmod\ 11)$ hay $x\equiv\ 2\ (\bmod\ 11)$

Thử lại thỏa mãn.

 

Câu 4:

1058702750_639123992_574_574.jpg

 

Không mất tổng quát giả sử $E$ nằm trên cung $BF.$

$a)$ $\widehat{ANC}=\textrm{sđ}\widehat{AC}+\textrm{sđ}\widehat{BE}=\textrm{sđ}\widehat{AB}+\textrm{sđ}\widehat{BE}=\textrm{sđ}\widehat{AE}=\widehat{EFE}$

Từ đó có điều phải chứng minh.

$b)$ Đường tròn tâm $I$ cắt đường tròn tâm $O$ tại $P.$

$AO$ cắt $EP$ tại $H,$ $OI$ cắt $PF$ tại $K.$

Ta có: 

$\widehat{FPD}=\widehat{FMD}=\widehat{FAE}=\widehat{FPE}$

Do đó $E, D, P$ thẳng hàng. Mà $ED\parallel BC$ nên $EP\parallel BC.$

Mặt khác $AO\perp BC$ nên $AO\perp EP \Rightarrow \widehat{OHP}=90^{\circ}$

Ta có: $OF=OP, IF=IP$ nên $OI$ là trung trực $PF.$

Nên $OI\perp PF \Rightarrow \widehat{OKP}=90^{\circ}$

Do đó tứ giác $OHKP$ nội tiếp $\Rightarrow \widehat{HOK}=\widehat{HPK}=\widehat{EAF}=\textrm{const}$

Từ đó suy ra tia $OI$ cố định. Vậy ta có điều phải chứng minh.

 

Spoiler




#487879 Cho $\Delta ABC$, $BD$ và $CE$ lần lượt là...

Đã gửi bởi DarkBlood on 19-03-2014 - 22:33 trong Hình học

Cho $\Delta ABC$, $BD$ và $CE$ lần lượt là 2 đường phân giác trong của tam giác tại đỉnh $B$ và $C$. Trên đoạn thẳng $DE$ lấy một điểm M bất kì.Từ $M$ kẻ các đường vuông góc với $BC,CA,BA$ lần lượt tại $I,J,K$. Chứng minh rằng trong ba đoạn thẳng $MI,MJ,MK$ có một đoạn thẳng bằng tổng hai đoạn còn lại.

Kẻ $DN \perp AB,\ EF \perp AC,\ EH \perp BC,\ DL\perp BC.$ 

Từ định lý Thales, suy ra

$MK+MJ=\dfrac{EM.DN+DM.EF}{DE}$

Mà $EF=EH,\ DN=DL$ 

Nên $MK+MJ=\dfrac{EM.DL+DM.EH}{DE}$

Gọi $O$ là giao điểm $EL$ với $MI.$

Theo định lý Thales, ta có

$OI=\dfrac{IL.EH}{LH}=\dfrac{DM.EH}{DE}$ và $MO=\dfrac{EM.DL}{DE}$

Suy ra $MI=OI+MO=\dfrac{EM.DL+DM.EH}{DE}$

Vậy $MI=MK+MJ$




#487024 Chứng minh rằng $CK \perp BN.$

Đã gửi bởi DarkBlood on 15-03-2014 - 20:34 trong Hình học

Cho hình vuông $ABCD$ có hai đường chéo cắt nhau tại $E.$ Một đường thẳng đi qua $A$ cắt cạnh $BC$ ở $M$ và cắt đường thẳng $CD$ ở $N.$ Gọi $K$ là giao điểm của $EM$ và $BN.$ Chứng minh rằng $CK \perp BN.$




#485738 $\dfrac{ka}{a^2+1}+\dfrac{5(a^2+1)...

Đã gửi bởi DarkBlood on 03-03-2014 - 23:42 trong Bất đẳng thức và cực trị

Tìm hằng số $k$ lớn nhất để bất đẳng thức $$\dfrac{ka}{a^2+1}+\dfrac{5(a^2+1)}{2a}\geq \dfrac{10+k}{2}$$ đúng với mọi số dương $a.$




#485732 Trận 4 - Bất đẳng thức

Đã gửi bởi DarkBlood on 03-03-2014 - 23:13 trong Thi giải toán Marathon cấp THCS 2014

Hình như những mở rộng của bạn buiminhhieu không liên quan tới bài toán thì phải  :mellow:




#485493 Trận 4 - Bất đẳng thức

Đã gửi bởi DarkBlood on 02-03-2014 - 17:45 trong Thi giải toán Marathon cấp THCS 2014

Cho $x,y\in \mathbb{R}$ thỏa mãn $(x+y)^{3}+4xy\geq 2$ . Tìm giá trị nhỏ nhất của:

$$ P=3(x^{4}+y^{4}+x^{2}y^{2})-2(x^{2}+y^{2})+1$$

Đề của 

nk0kckungtjnh

MSS 10

 

Mở rộng 1: Cho $x,y,z \in \mathbb{R}$ thỏa mãn $(x+y+z)^{3}+3(xy+yz+zx)\geq 2$ . Tìm giá trị nhỏ nhất của:

$$P=9(x^{4}+y^{4}+z^{4}+x^{2}y^{2}+y^{2}z^{2}+z^{2}x^{2})-3(x^{2}+y^{2}+z^{2})+2$$

 

Lời giải

Ta có $2\leq (x+y+z)^2+3(xy+yz+zx)\leq (x+y+z)^3+(x+y+z)^2$ 

Nên $(x+y+z)^3+(x+y+z)^2-2\geq 0 \Leftrightarrow (x+y+z-1)\left [(x+y+z)^2+2(x+y+z)+2 \right ]\geq 0$

Mà $(x+y+z)^2+2(x+y+z)+2>0$ nên $x+y+z-1\geq 0 \Leftrightarrow x+y+z\geq 1$

Mặt khác $x^2+y^2+z^2\geq \dfrac{1}{3}(x+y+z)^2$ nên $x^2+y^2+z^2\geq \dfrac{1}{3}.$

 

Ta có:

$P=9(x^{4}+y^{4}+z^{4}+x^{2}y^{2}+y^{2}z^{2}+z^{2}x^{2})-3(x^{2}+y^{2}+z^{2})+2$

$\Leftrightarrow P=9(x^2+y^2+z^2)^2-9(x^2y^2+y^2z^2+z^2x^2)-3(x^2+y^2+z^2)+2$

Mà $x^2y^2+y^2z^2+z^2x^2\leq \dfrac{1}{3}(x^2+y^2+z^2)^2$

Nên $P\geq 6(x^2+y^2+z^2)^2-3(x^2+y^2+z^2)+2=6\left [ (x^2+y^2+z^2)-\dfrac{1}{4} \right ]^2+\dfrac{13}{8}$

Vì $x^2+y^2+z^2\geq \dfrac{1}{3}$ nên $P\geq 6\left ( \dfrac{1}{3}-\dfrac{1}{4} \right )^2+\dfrac{13}{8}=\dfrac{5}{3}$

 

Vậy $\textrm{min}\ P=\dfrac{5}{3}$ khi $x=y=z=\dfrac{1}{3}.$

$$\star \star \star \star \star \star \star \star \star \star$$

Mở rộng 2: Cho $x,y\in \mathbb{R}$ thỏa mãn $(x+y)^{3}+4xy\geq 2$. Tìm giá trị nhỏ nhất của biểu thức sau với $n$ $(n\in \mathbb{N}^*)$ 

$$P=4\left ( x^{2^{n+1}}+y^{2^{n+1}}+x^{2^n}y^{2^n} \right )-\dfrac{6}{2^{2^n}}\left ( x^{2^n}+y^{2^n} \right )$$

 

Lời giải

Bổ đề Cho $x,y\in \mathbb{R}$ thỏa mãn $x+y\geq 1$ thì $x^{2^n}+y^{2^n}\geq \dfrac{1}{2^{2^{n}-1}}$ với mọi $n\in \mathbb{N}^*$ $\left ( \star \right )$

Chứng minh 

Với $n=1,$ ta có: $x^2+y^2\geq \dfrac{(x+y)^2}{2}\geq \dfrac{1}{2}=\dfrac{1}{2^{2^{1}-1}}$

Giả sử $\left ( \star \right )$ đúng với $n=k$ 

Khi đó $x^{2^k}+y^{2^k}\geq \dfrac{1}{2^{2^{k}-1}}$

Ta chứng minh với $n=k+1$ thì

$x^{2^{k+1}}+y^{2^{k+1}}\geq \dfrac{1}{2^{2^{k+1}-1}}$

Thật vậy, ta có

$x^{2^{k+1}}+y^{2^{k+1}}=\left ( x^{2^k} \right )^2+\left ( y^{2^k} \right )^2\geq \dfrac{1}{2}\left ( x^{2^k}+y^{2^k} \right )^2\geq \dfrac{1}{2}\left ( \dfrac{1}{2^{2^{k}-1}} \right )^2$

Mà $2\left (2^{2^{k}-1} \right )^2=2^{2^{k+1}-1}$ nên $x^{2^{k+1}}+y^{2^{k+1}}\geq \dfrac{1}{2^{2^{k+1}-1}}$

Vậy theo nguyên lý quy nạp, bổ đề được chứng minh.

 

Quay lại bài toán, từ lời giải bài toán gốc, ta được $x+y\geq 1$

Do đó $x^{2^n}+y^{2^n}\geq \dfrac{1}{2^{2^{n}-1}}$

Tương tự bài toán gốc, ta chứng minh được

$P\geq 3\left ( x^{2^n}+y^{2^n} \right )^2-\dfrac{6}{2^{2^n}}\left ( x^{2^n}+y^{2^n} \right )$

$\Leftrightarrow P\geq 3\left ( x^{2^n}+y^{2^n}-\dfrac{1}{2^{2^n}} \right )^2-\dfrac{3}{2^{2^{n+1}}}$

Mà $x^{2^n}+y^{2^n}\geq \dfrac{1}{2^{2^{n}-1}}$

Nên $P\geq 3\left ( \dfrac{1}{2^{2^{n}-1}}-\dfrac{1}{2^{2^n}} \right )^2-\dfrac{3}{2^{2^{n+1}}}=0$

 

Vậy $\textrm{min}\ P=0$ khi $x=y=\dfrac{1}{2}$

$$\star \star \star \star \star \star \star \star \star \star$$

Mở rộng 3: Giải hệ phương trình

$$\left\{\begin{matrix} x^3+y^3+3xy(x+y+2)=x^2+y^2+2 \\ 3(x^4+y^4+x^2y^2)−2(x^2+y^2)+\dfrac{7}{16}=0 \end{matrix}\right.$$

 

Lời giải

Từ phương trình ban đầu, ta có:

$(x+y)^3+4xy=2+(x-y)^2\geq 2$

Do đó $3(x^4+y^4+x^2y^2)−2(x^2+y^2)+1\geq \dfrac{9}{16}$ hay $3(x^4+y^4+x^2y^2)−2(x^2+y^2)+\dfrac{7}{16}\geq 0$

Đẳng thức xảy ra khi $x=y=\dfrac{1}{2}$

Do đó hệ phương trình có nghiệm $x=y=\dfrac{1}{2}.$

 

Có 4 nhận xét cho 3 bài mở rộng.

1) Anh ủng hộ em tìm tòi ra nhiều hướng như thế , nhưng .... ( đọc 2,3,4 )

2) MR 1 thì em không nên mở rộng bằng cách tạo ra thêm 1 ẩn ( nếu $n$ ẩn thì ok ) . Hãy tôn trọng ý tưởng người ra đề .

3 ) MR2 thì sai hoàn toàn . Định nghĩa hàm mũ là cơ số phải dương khác 1 .

4 ) Như 2 , em nên tông trọng dạng bài của tác giả

 

Điểm MR 0đ.